Kezdőoldal » Tudományok » Egyéb kérdések » Léteznek-e olyan nagy négyzet-...

Léteznek-e olyan nagy négyzet- és köbszámok, amelyek különbsége nagyon kicsi, de nem nulla?

Figyelt kérdés

Legyenek n,m > 2 egészek, k = n^2-m^3, k<>0 és n^2,m^3 > k^10

(Nem a 6. hatványok érdekelnek, ahol k=0)

Létezik ilyen kis különbség nagy négyzet- és köbszámok között?

(Pl. 11-nél kisebb különbség 10^10-nél nagyobb négyzet- és köbszámokkal, vagy 101-nél kisebb különbség 10^20-nál nagyobb négyzet- és köbszámokkal.)



2023. febr. 4. 19:43
 1/8 Kólauborkával ***** válasza:
2023. febr. 4. 20:07
Hasznos számodra ez a válasz?
 2/8 A kérdező kommentje:
És mi köze van az ikerprímeknek a négyzet- és köbszámokhoz?
2023. febr. 4. 22:20
 3/8 anonim ***** válasza:
Ez megint az okoskodó gyámhivatalos troll lesz.
2023. febr. 5. 00:12
Hasznos számodra ez a válasz?
 4/8 Kólauborkával ***** válasza:
O bocsánat valóban én néztem ezt most be. Ne haragudj.
2023. febr. 5. 00:23
Hasznos számodra ez a válasz?
 5/8 anonim ***** válasza:

Azt nem tudom, hogy nagyon-nagyon nagy számok esetén nincs-e ilyen szám, de írtam rá egy programot, ami ilyen számokat keres, de nem találtam olyan számot:


k = 1 és 10 között

n = 3 és 100 000 között

m = 3 és 100 000 között


Mivel nem ellenőriztem egyik esetben sem a túlcsordulást, nem lehet rá mérget venni, hogy a nagyobb számok esetén nem történt-e elszámolás, de több mint valószínűleg nincs.

2023. febr. 5. 15:50
Hasznos számodra ez a válasz?
 6/8 anonim ***** válasza:

public static void main(String[] args) {



for (int k = 1; k < 10; ++k) {


long k_10 = hatvany(k, 10);



System.out.println("k = " + k + " k¹⁰ = " + k_10);

for (long n = 3; n < 100000; ++n) {


for (long m = 3; m < 100000; ++m) {


long n_2 = hatvany(n, 2);

long m_3 = hatvany(m, 3);



if (n_2 - m_3 == k && n_2 > k_10 && m_3 > k_10) {

System.out.println("n = " + n + ", m = " + m + ", n² = " + n_2 + ", m³ = " + m_3);

}

}

}



System.out.println("nincs");

}


System.out.println("vége");


}



Itt a program is.

2023. febr. 5. 15:51
Hasznos számodra ez a válasz?
 7/8 anonim ***** válasza:

Megkérdeztem a robotot is, aki a következőt válaszolta:


Nem létezik olyan n és m szám, ami megfelelne a fenti feltételnek. Ha n² és m³ nagyon nagy számok, akkor n² - m³ is nagyon nagy szám lesz, ami nem lehet kisebb mint k¹⁰. Tehát nem lehetséges, hogy n² - m³ = k, ahol k < k¹⁰.



Amikor megkértem, hogy bizonyítsa, ezt írta:



Igen, a bizonyítás során használhatjuk a matematikai induction elvét. Az alábbi lépésekkel bizonyíthatjuk:


Vegyük figyelembe az állítás alap esetét, amikor n = m = 3. Ebben az esetben n² - m³ = 9 - 27 = -18, ami kisebb mint k¹⁰ amikor k = 10.


Feltételezzük, hogy n = m = k esetén n² - m³ < k¹⁰. Azaz (k+1)² - (k+1)³ < (k+1)¹⁰.


Most bizonyítsuk el, hogy ha az alap eset és a feltétel igaz, akkor k+1 esetén is igaz lesz. Azaz (k+1)² - (k+1)³ < (k+1)¹⁰.


Az (k+1)² - (k+1)³ = k² + 2k + 1 - (k³ + 3k² + 3k + 1) = k² - k³ + 2k < k¹⁰ + 2k < (k+1)¹⁰.


Tehát ha az alap eset és a feltétel igaz, akkor a k+1 eset is igaz lesz.


Mivel az alap eset igaz, azt mondhatjuk, hogy minden n = m = k esetén n² - m³ < k¹⁰.


Tehát az állítás minden n és m esetére igaz, amikor n = m > 2, tehát n² - m³ nem lehet nem nulla egész szám, amikor n², m³ > k¹⁰.


Mivel a matematika nem a legfőbb szakterületem, nem tudom, hogy ez a bizonyítás megállja a helyét, de a robot csak jobban tudja mint én, és gondolom lesznek itt matematikusok is, akik ellenőrizhetik.

2023. febr. 5. 15:57
Hasznos számodra ez a válasz?
 8/8 A kérdező kommentje:

Ha valóban ezt írta:

"Feltételezzük, hogy n = m = k esetén n² - m³ < k¹⁰. Azaz (k+1)² - (k+1)³ < (k+1)¹⁰."

akkor hülyeség, hiszen nagyságrendileg különböző n-t, és m-t keresünk!

Pl. ha n 10^9 körül van, akkor m-nek 10^6 körülinek kell lennie. (2*9=3*6)

A programhoz:

Nem kell kettős ciklus!

Adott m-hez ki kell számítani a megfelelő n-t a köv. módon:

n' = gyök(m^3) = m*gyök(m) = m^1.5

n pedig n' legközelebbi egészre kerekítése.

Mivel n^2 mellett a szomszédos négyzetszámok 2n-1 vagy 2n+1 távolságra vannak, a k értékek nagyjából véletlenszerűek lesznek a 0..n intervallumban.

(De n nagyon gyorsan nő, m^1.5 szerint). Tehát egy igen gyorsan növekvő intervallumban lesz-e egy igen kicsi, nem 0 érték?

Néhány m, n, k=m^3-n^2:

5001 353659 326720 | 5002 353766 -322748 | 5003 353872 -257357,

5004 353978 -184420 | 5005 354084 -103931 | 5006 354190 -15884

2023. febr. 5. 18:13

Kapcsolódó kérdések:





Minden jog fenntartva © 2024, www.gyakorikerdesek.hu
GYIK | Szabályzat | Jogi nyilatkozat | Adatvédelem | Cookie beállítások | WebMinute Kft. | Facebook | Kapcsolat: info(kukac)gyakorikerdesek.hu

A weboldalon megjelenő anyagok nem minősülnek szerkesztői tartalomnak, előzetes ellenőrzésen nem esnek át, az üzemeltető véleményét nem tükrözik.
Ha kifogással szeretne élni valamely tartalommal kapcsolatban, kérjük jelezze e-mailes elérhetőségünkön!